Are there examples of conjectures supported by heuristic arguments that have been finally disproved?

The name of the pictureThe name of the pictureThe name of the pictureClash Royale CLAN TAG#URR8PPP











up vote
45
down vote

favorite
16












The idea for this comes from the twin prime conjecture, where the heuristic evidence seems just so overwhelming, especially in the light of Zhang's famous result from 2014 about Bounded gaps between primes and its subsequent improvements.




Are there examples of conjectures with some more or less "good" heuristic arguments, but where those arguments have finally not been "strong" enough?




I don't mean heuristic in the sense that some conjecture holds for small numbers, e.g. the fact that $li x-pi (x) $ was thought to be always positive,
before Littlewood showed that not only it eventually changes sign, but does so infinitely often later on. So my question is different from the question about eventual counterexamples.



Likewise, the fact that the first $10^15$ or so zeros of the zeta function "obey" RH does tell us something, but not a whole lot compared with infinity. So again this is not what I mean by heuristic.










share|cite|improve this question



















  • 4




    en.wikipedia.org/wiki/List_of_disproved_mathematical_ideas
    – Steve Huntsman
    Aug 16 at 18:44






  • 4




    The question in your title seems subtly different than the question in the box. Is Maier's theorem an example of what you are looking for? Cramér's heuristic breaks down here.
    – Mark S
    Aug 16 at 18:58







  • 5




    See also mathoverflow.net/questions/30149
    – j.c.
    Aug 16 at 22:01






  • 4




    @MarkS : Why not post Maier's theorem as an answer?
    – Timothy Chow
    Aug 17 at 1:26






  • 4




    @JoelDavidHamkins I guess that is just the MO policy/tradition for "big-list" questions. Now of course you, more than anybody else here, may feel free to use your privilege of "un-CW-ing" what some other mod made CW. But anyway, supposedly not many people will post an answer only because of getting some virtual reward for that. Sure I agree that such a reward is always nice for one's ego, but there are rewards which are far more precious in life. :) :)
    – Wolfgang
    Aug 17 at 9:36















up vote
45
down vote

favorite
16












The idea for this comes from the twin prime conjecture, where the heuristic evidence seems just so overwhelming, especially in the light of Zhang's famous result from 2014 about Bounded gaps between primes and its subsequent improvements.




Are there examples of conjectures with some more or less "good" heuristic arguments, but where those arguments have finally not been "strong" enough?




I don't mean heuristic in the sense that some conjecture holds for small numbers, e.g. the fact that $li x-pi (x) $ was thought to be always positive,
before Littlewood showed that not only it eventually changes sign, but does so infinitely often later on. So my question is different from the question about eventual counterexamples.



Likewise, the fact that the first $10^15$ or so zeros of the zeta function "obey" RH does tell us something, but not a whole lot compared with infinity. So again this is not what I mean by heuristic.










share|cite|improve this question



















  • 4




    en.wikipedia.org/wiki/List_of_disproved_mathematical_ideas
    – Steve Huntsman
    Aug 16 at 18:44






  • 4




    The question in your title seems subtly different than the question in the box. Is Maier's theorem an example of what you are looking for? Cramér's heuristic breaks down here.
    – Mark S
    Aug 16 at 18:58







  • 5




    See also mathoverflow.net/questions/30149
    – j.c.
    Aug 16 at 22:01






  • 4




    @MarkS : Why not post Maier's theorem as an answer?
    – Timothy Chow
    Aug 17 at 1:26






  • 4




    @JoelDavidHamkins I guess that is just the MO policy/tradition for "big-list" questions. Now of course you, more than anybody else here, may feel free to use your privilege of "un-CW-ing" what some other mod made CW. But anyway, supposedly not many people will post an answer only because of getting some virtual reward for that. Sure I agree that such a reward is always nice for one's ego, but there are rewards which are far more precious in life. :) :)
    – Wolfgang
    Aug 17 at 9:36













up vote
45
down vote

favorite
16









up vote
45
down vote

favorite
16






16





The idea for this comes from the twin prime conjecture, where the heuristic evidence seems just so overwhelming, especially in the light of Zhang's famous result from 2014 about Bounded gaps between primes and its subsequent improvements.




Are there examples of conjectures with some more or less "good" heuristic arguments, but where those arguments have finally not been "strong" enough?




I don't mean heuristic in the sense that some conjecture holds for small numbers, e.g. the fact that $li x-pi (x) $ was thought to be always positive,
before Littlewood showed that not only it eventually changes sign, but does so infinitely often later on. So my question is different from the question about eventual counterexamples.



Likewise, the fact that the first $10^15$ or so zeros of the zeta function "obey" RH does tell us something, but not a whole lot compared with infinity. So again this is not what I mean by heuristic.










share|cite|improve this question















The idea for this comes from the twin prime conjecture, where the heuristic evidence seems just so overwhelming, especially in the light of Zhang's famous result from 2014 about Bounded gaps between primes and its subsequent improvements.




Are there examples of conjectures with some more or less "good" heuristic arguments, but where those arguments have finally not been "strong" enough?




I don't mean heuristic in the sense that some conjecture holds for small numbers, e.g. the fact that $li x-pi (x) $ was thought to be always positive,
before Littlewood showed that not only it eventually changes sign, but does so infinitely often later on. So my question is different from the question about eventual counterexamples.



Likewise, the fact that the first $10^15$ or so zeros of the zeta function "obey" RH does tell us something, but not a whole lot compared with infinity. So again this is not what I mean by heuristic.







nt.number-theory big-list conjectures






share|cite|improve this question















share|cite|improve this question













share|cite|improve this question




share|cite|improve this question








edited Aug 17 at 11:51


























community wiki





2 revs, 2 users 100%
Wolfgang








  • 4




    en.wikipedia.org/wiki/List_of_disproved_mathematical_ideas
    – Steve Huntsman
    Aug 16 at 18:44






  • 4




    The question in your title seems subtly different than the question in the box. Is Maier's theorem an example of what you are looking for? Cramér's heuristic breaks down here.
    – Mark S
    Aug 16 at 18:58







  • 5




    See also mathoverflow.net/questions/30149
    – j.c.
    Aug 16 at 22:01






  • 4




    @MarkS : Why not post Maier's theorem as an answer?
    – Timothy Chow
    Aug 17 at 1:26






  • 4




    @JoelDavidHamkins I guess that is just the MO policy/tradition for "big-list" questions. Now of course you, more than anybody else here, may feel free to use your privilege of "un-CW-ing" what some other mod made CW. But anyway, supposedly not many people will post an answer only because of getting some virtual reward for that. Sure I agree that such a reward is always nice for one's ego, but there are rewards which are far more precious in life. :) :)
    – Wolfgang
    Aug 17 at 9:36













  • 4




    en.wikipedia.org/wiki/List_of_disproved_mathematical_ideas
    – Steve Huntsman
    Aug 16 at 18:44






  • 4




    The question in your title seems subtly different than the question in the box. Is Maier's theorem an example of what you are looking for? Cramér's heuristic breaks down here.
    – Mark S
    Aug 16 at 18:58







  • 5




    See also mathoverflow.net/questions/30149
    – j.c.
    Aug 16 at 22:01






  • 4




    @MarkS : Why not post Maier's theorem as an answer?
    – Timothy Chow
    Aug 17 at 1:26






  • 4




    @JoelDavidHamkins I guess that is just the MO policy/tradition for "big-list" questions. Now of course you, more than anybody else here, may feel free to use your privilege of "un-CW-ing" what some other mod made CW. But anyway, supposedly not many people will post an answer only because of getting some virtual reward for that. Sure I agree that such a reward is always nice for one's ego, but there are rewards which are far more precious in life. :) :)
    – Wolfgang
    Aug 17 at 9:36








4




4




en.wikipedia.org/wiki/List_of_disproved_mathematical_ideas
– Steve Huntsman
Aug 16 at 18:44




en.wikipedia.org/wiki/List_of_disproved_mathematical_ideas
– Steve Huntsman
Aug 16 at 18:44




4




4




The question in your title seems subtly different than the question in the box. Is Maier's theorem an example of what you are looking for? Cramér's heuristic breaks down here.
– Mark S
Aug 16 at 18:58





The question in your title seems subtly different than the question in the box. Is Maier's theorem an example of what you are looking for? Cramér's heuristic breaks down here.
– Mark S
Aug 16 at 18:58





5




5




See also mathoverflow.net/questions/30149
– j.c.
Aug 16 at 22:01




See also mathoverflow.net/questions/30149
– j.c.
Aug 16 at 22:01




4




4




@MarkS : Why not post Maier's theorem as an answer?
– Timothy Chow
Aug 17 at 1:26




@MarkS : Why not post Maier's theorem as an answer?
– Timothy Chow
Aug 17 at 1:26




4




4




@JoelDavidHamkins I guess that is just the MO policy/tradition for "big-list" questions. Now of course you, more than anybody else here, may feel free to use your privilege of "un-CW-ing" what some other mod made CW. But anyway, supposedly not many people will post an answer only because of getting some virtual reward for that. Sure I agree that such a reward is always nice for one's ego, but there are rewards which are far more precious in life. :) :)
– Wolfgang
Aug 17 at 9:36





@JoelDavidHamkins I guess that is just the MO policy/tradition for "big-list" questions. Now of course you, more than anybody else here, may feel free to use your privilege of "un-CW-ing" what some other mod made CW. But anyway, supposedly not many people will post an answer only because of getting some virtual reward for that. Sure I agree that such a reward is always nice for one's ego, but there are rewards which are far more precious in life. :) :)
– Wolfgang
Aug 17 at 9:36











6 Answers
6






active

oldest

votes

















up vote
31
down vote













In computational complexity theory, most conjectures that two complexity classes are equal (or not equal, as the case may be) can be relativized to an oracle. Sometimes, as in the case of P = NP, one can obtain contradictory relativizations; i.e., there exists an oracle A such that PA = NPA and an oracle B such that PB ≠ NPB.



In the case of contradictory relativizations, it is tempting to hypothesize that if, for example, PB ≠ NPB for "most" oracles B, then P ≠ NP in the "real" (unrelativized) world. This heuristic was seriously proposed by Bennett and Gill as the "random oracle hypothesis," for a specific precise definition of "most oracles." However, the random oracle hypothesis was disproved by Kurtz. Later, another conjecture was proposed along similar lines: the "generic oracle hypothesis," with a different precise definition of "most oracles." But the generic oracle hypothesis was also disproved, by Foster.






share|cite|improve this answer






















  • What is different between this and recent result polynomial hierarchy is infinite with respect to a random oracle result?
    – Freeman.
    Aug 17 at 12:36






  • 3




    In 2015, Rossman, Servedio, Tan strengthened Yao and Hastad's theorem (PH is infinite relative to some oracle) to "PH is infinite relative to a random oracle." Because the random oracle hypothesis is false in general, the Rossman et al. theorem does not provide as strong evidence for "PH is infinite" as it might otherwise, but it is still a stronger result than Yao and Hastad's theorem.
    – Timothy Chow
    Aug 17 at 14:07

















up vote
26
down vote













This is extremely common in the field of cryptography. It was popular to design key exchange systems in the 1980s without proof of security, just giving intuitive arguments which were often quite convincing. It turns out a lot of these systems became broken later.



Distributed systems are difficult to have good intuition about because they are complicated objects. However, we have a lot of (wrong) intuition around them because they are physical systems and we think we understand them. Hence, it is easy to make mistakes. There are many cases where systems work contrary to intuition.



One of my favourite examples is bitcoin. The original paper gives an argument about why the system is secure and why the honest strategy is a Nash equilibrium, by illustrating that one particular attack doesn't work in favour of an adversary. This is more than just a hand-wavy argument: It is the full calculation of a particular attack, analytically and with numbers. Furthermore, that particular attack seems the most reasonable thing to do.



However, it was later shown that there are strategies which are better than the honest strategy -- the "selfish mining" attacks. The latter paper describes an alternative attack, quite more complicated, than what the original author envisioned. The attack is contrary to intuition. The bitcoin protocol was later fully analyzed on the backbone paper and it was proven secure for any adversarial strategy. However, the selfish mining strategy remains a better strategy than the honest strategy (and in fact the backbone paper shows a tight bound on this class of attacks), so bitcoin is not incentive-compatible.



Another example around bitcoin-related conjectures is that, as time goes by and coinbase rewards are decreased, fees will make up for the incentives to continue running the blockchain. This notion is so ingrained in the bitcoin community that the bitcoin wiki literally states that "In the future, as the number of new bitcoins miners are allowed to create in each block dwindles, the fees will make up a much more important percentage of mining income." This conjecture has been shown to be false.






share|cite|improve this answer


















  • 8




    I read recently someone's observation that it is extremely easy for one to conceive of cryptosystems that oneself could never conceive of breaking. Truer words have never BitToken.
    – PrimeRibeyeDeal
    Aug 17 at 17:23










  • Groan! (which is high praise for a pun)
    – Brian Hopkins
    Aug 20 at 14:15


















up vote
21
down vote













I think the OP is looking for answers where there is a heuristic justification, apart from simply a large amount of numerical evidence, that in some instances may break down. For example, "we morally expect objects to behave a certain way, but in this instance, they don't behave quite the way we expect them to."



Cramér's probabilistic model, from the 1930's, is a powerful heuristic for, among others, estimating gaps between prime numbers.



However, Maier's theorem from 1985 states that, for all $lambdagt 1$,



$$fracpi(x+(log x)^lambda)-pi(x)(log x)^lambda-1$$



does not have a limit as $x$ goes to infinity, whereas following the Cramér heuristic, one would have a limit of $1$ for all $lambda gt 2$.



Pintz revisits Maier's theorem, studying weaknesses of such probabilistic models with technology available in the '30s. He also hints that there are ways to improve the CM heuristic to correctly predict Maier's limit, but goes on to provide another weakness inherent to all such models that, although smaller, "seems impossible to correct."






share|cite|improve this answer


















  • 3




    Is there an improved heuristic model that predicts Maier's theorem?
    – YCor
    Aug 17 at 21:08

















up vote
18
down vote













It has been conjectured that, for all $n$, there is no interval of length $n$ with more primes in it than the interval between $2$ and $n+1$. You look at a table of primes, and you see how they thin out the higher up you go, and that's evidence, of a sort. But more precisely, we know that the density of the primes among the first $n$ numbers goes to zero as $n$ goes to infinity, so that seems like a heuristic supporting the conjecture.



And the conjecture hasn't actually been disproved, but some 40 years ago, Hensley & Richards proved it contradicted the prime $k$-tuples conjecture, which has what's considered to be stronger supporting evidence. So at least one of two conjectures with heuristic support is false, we just don't have a decision yet on which one.






share|cite|improve this answer


















  • 3




    Off topic, but I was struck by the line in the linked paper: "It was necessary to go into machine language in order to handle a sequence of 10^5 bits without taking too much computer time and memory space". How things change...
    – Yemon Choi
    Aug 18 at 17:06










  • Maybe this decreasing prime density conjecture is false just because you encounter more prime powers in [2,n+1] than in other intervals of length n : the prime powers 4, 8, 9, 16, 25, 27, 32, 49 all appear before 50 so it may not be that surprising that some translate of such an interval actually contains more primes.
    – Sylvain JULIEN
    Aug 18 at 17:33











  • And as a matter of fact, adding 100 to the previous numbers yields the three primes 109, 127 and 149.
    – Sylvain JULIEN
    Aug 18 at 17:38

















up vote
13
down vote













Ruelle's "heuristic theory of phase transitions" (Comm. Math. Phys., Volume 53, Number 3 (1977), 195-208) turned out to be false in certain Banach spaces of interactions: see my "Generic triviality of phase diagrams in spaces of long-range interactions" (Comm. Math. Phys., Volume 106, Number 3 (1986), 459-466).






share|cite|improve this answer





























    up vote
    4
    down vote













    I think Hauptvermutung (the "main conjecture" in German) is a good example.
    It certainly is supported by very plausible heuristic arguments, and nobody had any doubt for half a century.






    share|cite|improve this answer






















    • What are those "very plausible heuristic arguments"?
      – Wojowu
      Aug 18 at 10:28










    • Edwin Moise in his well known paper "Affine structures ..." (Annals of Math,1952) for this sort of arguments makes a reference to "Zur Topologie der Mannigfaltigkeiten", G Nöbeling, Monatshefte für Mathematik und Physik, 1935. I can't say much more for I could not read it even if I had it (I am not very good at German). On the undergrad level, it is plausible at least for manifolds because continuous maps can be approximated by piecewise linear ones. Of course, today we know that this logic is wrong.
      – Alex Gavrilov
      Aug 18 at 11:10






    • 3




      mathoverflow.net/questions/51531/…
      – Alex Gavrilov
      Aug 18 at 11:25










    Your Answer




    StackExchange.ifUsing("editor", function ()
    return StackExchange.using("mathjaxEditing", function ()
    StackExchange.MarkdownEditor.creationCallbacks.add(function (editor, postfix)
    StackExchange.mathjaxEditing.prepareWmdForMathJax(editor, postfix, [["$", "$"], ["\\(","\\)"]]);
    );
    );
    , "mathjax-editing");

    StackExchange.ready(function()
    var channelOptions =
    tags: "".split(" "),
    id: "504"
    ;
    initTagRenderer("".split(" "), "".split(" "), channelOptions);

    StackExchange.using("externalEditor", function()
    // Have to fire editor after snippets, if snippets enabled
    if (StackExchange.settings.snippets.snippetsEnabled)
    StackExchange.using("snippets", function()
    createEditor();
    );

    else
    createEditor();

    );

    function createEditor()
    StackExchange.prepareEditor(
    heartbeatType: 'answer',
    convertImagesToLinks: true,
    noModals: false,
    showLowRepImageUploadWarning: true,
    reputationToPostImages: 10,
    bindNavPrevention: true,
    postfix: "",
    noCode: true, onDemand: true,
    discardSelector: ".discard-answer"
    ,immediatelyShowMarkdownHelp:true
    );



    );













     

    draft saved


    draft discarded


















    StackExchange.ready(
    function ()
    StackExchange.openid.initPostLogin('.new-post-login', 'https%3a%2f%2fmathoverflow.net%2fquestions%2f308464%2fare-there-examples-of-conjectures-supported-by-heuristic-arguments-that-have-bee%23new-answer', 'question_page');

    );

    Post as a guest






























    6 Answers
    6






    active

    oldest

    votes








    6 Answers
    6






    active

    oldest

    votes









    active

    oldest

    votes






    active

    oldest

    votes








    up vote
    31
    down vote













    In computational complexity theory, most conjectures that two complexity classes are equal (or not equal, as the case may be) can be relativized to an oracle. Sometimes, as in the case of P = NP, one can obtain contradictory relativizations; i.e., there exists an oracle A such that PA = NPA and an oracle B such that PB ≠ NPB.



    In the case of contradictory relativizations, it is tempting to hypothesize that if, for example, PB ≠ NPB for "most" oracles B, then P ≠ NP in the "real" (unrelativized) world. This heuristic was seriously proposed by Bennett and Gill as the "random oracle hypothesis," for a specific precise definition of "most oracles." However, the random oracle hypothesis was disproved by Kurtz. Later, another conjecture was proposed along similar lines: the "generic oracle hypothesis," with a different precise definition of "most oracles." But the generic oracle hypothesis was also disproved, by Foster.






    share|cite|improve this answer






















    • What is different between this and recent result polynomial hierarchy is infinite with respect to a random oracle result?
      – Freeman.
      Aug 17 at 12:36






    • 3




      In 2015, Rossman, Servedio, Tan strengthened Yao and Hastad's theorem (PH is infinite relative to some oracle) to "PH is infinite relative to a random oracle." Because the random oracle hypothesis is false in general, the Rossman et al. theorem does not provide as strong evidence for "PH is infinite" as it might otherwise, but it is still a stronger result than Yao and Hastad's theorem.
      – Timothy Chow
      Aug 17 at 14:07














    up vote
    31
    down vote













    In computational complexity theory, most conjectures that two complexity classes are equal (or not equal, as the case may be) can be relativized to an oracle. Sometimes, as in the case of P = NP, one can obtain contradictory relativizations; i.e., there exists an oracle A such that PA = NPA and an oracle B such that PB ≠ NPB.



    In the case of contradictory relativizations, it is tempting to hypothesize that if, for example, PB ≠ NPB for "most" oracles B, then P ≠ NP in the "real" (unrelativized) world. This heuristic was seriously proposed by Bennett and Gill as the "random oracle hypothesis," for a specific precise definition of "most oracles." However, the random oracle hypothesis was disproved by Kurtz. Later, another conjecture was proposed along similar lines: the "generic oracle hypothesis," with a different precise definition of "most oracles." But the generic oracle hypothesis was also disproved, by Foster.






    share|cite|improve this answer






















    • What is different between this and recent result polynomial hierarchy is infinite with respect to a random oracle result?
      – Freeman.
      Aug 17 at 12:36






    • 3




      In 2015, Rossman, Servedio, Tan strengthened Yao and Hastad's theorem (PH is infinite relative to some oracle) to "PH is infinite relative to a random oracle." Because the random oracle hypothesis is false in general, the Rossman et al. theorem does not provide as strong evidence for "PH is infinite" as it might otherwise, but it is still a stronger result than Yao and Hastad's theorem.
      – Timothy Chow
      Aug 17 at 14:07












    up vote
    31
    down vote










    up vote
    31
    down vote









    In computational complexity theory, most conjectures that two complexity classes are equal (or not equal, as the case may be) can be relativized to an oracle. Sometimes, as in the case of P = NP, one can obtain contradictory relativizations; i.e., there exists an oracle A such that PA = NPA and an oracle B such that PB ≠ NPB.



    In the case of contradictory relativizations, it is tempting to hypothesize that if, for example, PB ≠ NPB for "most" oracles B, then P ≠ NP in the "real" (unrelativized) world. This heuristic was seriously proposed by Bennett and Gill as the "random oracle hypothesis," for a specific precise definition of "most oracles." However, the random oracle hypothesis was disproved by Kurtz. Later, another conjecture was proposed along similar lines: the "generic oracle hypothesis," with a different precise definition of "most oracles." But the generic oracle hypothesis was also disproved, by Foster.






    share|cite|improve this answer














    In computational complexity theory, most conjectures that two complexity classes are equal (or not equal, as the case may be) can be relativized to an oracle. Sometimes, as in the case of P = NP, one can obtain contradictory relativizations; i.e., there exists an oracle A such that PA = NPA and an oracle B such that PB ≠ NPB.



    In the case of contradictory relativizations, it is tempting to hypothesize that if, for example, PB ≠ NPB for "most" oracles B, then P ≠ NP in the "real" (unrelativized) world. This heuristic was seriously proposed by Bennett and Gill as the "random oracle hypothesis," for a specific precise definition of "most oracles." However, the random oracle hypothesis was disproved by Kurtz. Later, another conjecture was proposed along similar lines: the "generic oracle hypothesis," with a different precise definition of "most oracles." But the generic oracle hypothesis was also disproved, by Foster.







    share|cite|improve this answer














    share|cite|improve this answer



    share|cite|improve this answer








    answered Aug 16 at 22:12


























    community wiki





    Timothy Chow












    • What is different between this and recent result polynomial hierarchy is infinite with respect to a random oracle result?
      – Freeman.
      Aug 17 at 12:36






    • 3




      In 2015, Rossman, Servedio, Tan strengthened Yao and Hastad's theorem (PH is infinite relative to some oracle) to "PH is infinite relative to a random oracle." Because the random oracle hypothesis is false in general, the Rossman et al. theorem does not provide as strong evidence for "PH is infinite" as it might otherwise, but it is still a stronger result than Yao and Hastad's theorem.
      – Timothy Chow
      Aug 17 at 14:07
















    • What is different between this and recent result polynomial hierarchy is infinite with respect to a random oracle result?
      – Freeman.
      Aug 17 at 12:36






    • 3




      In 2015, Rossman, Servedio, Tan strengthened Yao and Hastad's theorem (PH is infinite relative to some oracle) to "PH is infinite relative to a random oracle." Because the random oracle hypothesis is false in general, the Rossman et al. theorem does not provide as strong evidence for "PH is infinite" as it might otherwise, but it is still a stronger result than Yao and Hastad's theorem.
      – Timothy Chow
      Aug 17 at 14:07















    What is different between this and recent result polynomial hierarchy is infinite with respect to a random oracle result?
    – Freeman.
    Aug 17 at 12:36




    What is different between this and recent result polynomial hierarchy is infinite with respect to a random oracle result?
    – Freeman.
    Aug 17 at 12:36




    3




    3




    In 2015, Rossman, Servedio, Tan strengthened Yao and Hastad's theorem (PH is infinite relative to some oracle) to "PH is infinite relative to a random oracle." Because the random oracle hypothesis is false in general, the Rossman et al. theorem does not provide as strong evidence for "PH is infinite" as it might otherwise, but it is still a stronger result than Yao and Hastad's theorem.
    – Timothy Chow
    Aug 17 at 14:07




    In 2015, Rossman, Servedio, Tan strengthened Yao and Hastad's theorem (PH is infinite relative to some oracle) to "PH is infinite relative to a random oracle." Because the random oracle hypothesis is false in general, the Rossman et al. theorem does not provide as strong evidence for "PH is infinite" as it might otherwise, but it is still a stronger result than Yao and Hastad's theorem.
    – Timothy Chow
    Aug 17 at 14:07










    up vote
    26
    down vote













    This is extremely common in the field of cryptography. It was popular to design key exchange systems in the 1980s without proof of security, just giving intuitive arguments which were often quite convincing. It turns out a lot of these systems became broken later.



    Distributed systems are difficult to have good intuition about because they are complicated objects. However, we have a lot of (wrong) intuition around them because they are physical systems and we think we understand them. Hence, it is easy to make mistakes. There are many cases where systems work contrary to intuition.



    One of my favourite examples is bitcoin. The original paper gives an argument about why the system is secure and why the honest strategy is a Nash equilibrium, by illustrating that one particular attack doesn't work in favour of an adversary. This is more than just a hand-wavy argument: It is the full calculation of a particular attack, analytically and with numbers. Furthermore, that particular attack seems the most reasonable thing to do.



    However, it was later shown that there are strategies which are better than the honest strategy -- the "selfish mining" attacks. The latter paper describes an alternative attack, quite more complicated, than what the original author envisioned. The attack is contrary to intuition. The bitcoin protocol was later fully analyzed on the backbone paper and it was proven secure for any adversarial strategy. However, the selfish mining strategy remains a better strategy than the honest strategy (and in fact the backbone paper shows a tight bound on this class of attacks), so bitcoin is not incentive-compatible.



    Another example around bitcoin-related conjectures is that, as time goes by and coinbase rewards are decreased, fees will make up for the incentives to continue running the blockchain. This notion is so ingrained in the bitcoin community that the bitcoin wiki literally states that "In the future, as the number of new bitcoins miners are allowed to create in each block dwindles, the fees will make up a much more important percentage of mining income." This conjecture has been shown to be false.






    share|cite|improve this answer


















    • 8




      I read recently someone's observation that it is extremely easy for one to conceive of cryptosystems that oneself could never conceive of breaking. Truer words have never BitToken.
      – PrimeRibeyeDeal
      Aug 17 at 17:23










    • Groan! (which is high praise for a pun)
      – Brian Hopkins
      Aug 20 at 14:15















    up vote
    26
    down vote













    This is extremely common in the field of cryptography. It was popular to design key exchange systems in the 1980s without proof of security, just giving intuitive arguments which were often quite convincing. It turns out a lot of these systems became broken later.



    Distributed systems are difficult to have good intuition about because they are complicated objects. However, we have a lot of (wrong) intuition around them because they are physical systems and we think we understand them. Hence, it is easy to make mistakes. There are many cases where systems work contrary to intuition.



    One of my favourite examples is bitcoin. The original paper gives an argument about why the system is secure and why the honest strategy is a Nash equilibrium, by illustrating that one particular attack doesn't work in favour of an adversary. This is more than just a hand-wavy argument: It is the full calculation of a particular attack, analytically and with numbers. Furthermore, that particular attack seems the most reasonable thing to do.



    However, it was later shown that there are strategies which are better than the honest strategy -- the "selfish mining" attacks. The latter paper describes an alternative attack, quite more complicated, than what the original author envisioned. The attack is contrary to intuition. The bitcoin protocol was later fully analyzed on the backbone paper and it was proven secure for any adversarial strategy. However, the selfish mining strategy remains a better strategy than the honest strategy (and in fact the backbone paper shows a tight bound on this class of attacks), so bitcoin is not incentive-compatible.



    Another example around bitcoin-related conjectures is that, as time goes by and coinbase rewards are decreased, fees will make up for the incentives to continue running the blockchain. This notion is so ingrained in the bitcoin community that the bitcoin wiki literally states that "In the future, as the number of new bitcoins miners are allowed to create in each block dwindles, the fees will make up a much more important percentage of mining income." This conjecture has been shown to be false.






    share|cite|improve this answer


















    • 8




      I read recently someone's observation that it is extremely easy for one to conceive of cryptosystems that oneself could never conceive of breaking. Truer words have never BitToken.
      – PrimeRibeyeDeal
      Aug 17 at 17:23










    • Groan! (which is high praise for a pun)
      – Brian Hopkins
      Aug 20 at 14:15













    up vote
    26
    down vote










    up vote
    26
    down vote









    This is extremely common in the field of cryptography. It was popular to design key exchange systems in the 1980s without proof of security, just giving intuitive arguments which were often quite convincing. It turns out a lot of these systems became broken later.



    Distributed systems are difficult to have good intuition about because they are complicated objects. However, we have a lot of (wrong) intuition around them because they are physical systems and we think we understand them. Hence, it is easy to make mistakes. There are many cases where systems work contrary to intuition.



    One of my favourite examples is bitcoin. The original paper gives an argument about why the system is secure and why the honest strategy is a Nash equilibrium, by illustrating that one particular attack doesn't work in favour of an adversary. This is more than just a hand-wavy argument: It is the full calculation of a particular attack, analytically and with numbers. Furthermore, that particular attack seems the most reasonable thing to do.



    However, it was later shown that there are strategies which are better than the honest strategy -- the "selfish mining" attacks. The latter paper describes an alternative attack, quite more complicated, than what the original author envisioned. The attack is contrary to intuition. The bitcoin protocol was later fully analyzed on the backbone paper and it was proven secure for any adversarial strategy. However, the selfish mining strategy remains a better strategy than the honest strategy (and in fact the backbone paper shows a tight bound on this class of attacks), so bitcoin is not incentive-compatible.



    Another example around bitcoin-related conjectures is that, as time goes by and coinbase rewards are decreased, fees will make up for the incentives to continue running the blockchain. This notion is so ingrained in the bitcoin community that the bitcoin wiki literally states that "In the future, as the number of new bitcoins miners are allowed to create in each block dwindles, the fees will make up a much more important percentage of mining income." This conjecture has been shown to be false.






    share|cite|improve this answer














    This is extremely common in the field of cryptography. It was popular to design key exchange systems in the 1980s without proof of security, just giving intuitive arguments which were often quite convincing. It turns out a lot of these systems became broken later.



    Distributed systems are difficult to have good intuition about because they are complicated objects. However, we have a lot of (wrong) intuition around them because they are physical systems and we think we understand them. Hence, it is easy to make mistakes. There are many cases where systems work contrary to intuition.



    One of my favourite examples is bitcoin. The original paper gives an argument about why the system is secure and why the honest strategy is a Nash equilibrium, by illustrating that one particular attack doesn't work in favour of an adversary. This is more than just a hand-wavy argument: It is the full calculation of a particular attack, analytically and with numbers. Furthermore, that particular attack seems the most reasonable thing to do.



    However, it was later shown that there are strategies which are better than the honest strategy -- the "selfish mining" attacks. The latter paper describes an alternative attack, quite more complicated, than what the original author envisioned. The attack is contrary to intuition. The bitcoin protocol was later fully analyzed on the backbone paper and it was proven secure for any adversarial strategy. However, the selfish mining strategy remains a better strategy than the honest strategy (and in fact the backbone paper shows a tight bound on this class of attacks), so bitcoin is not incentive-compatible.



    Another example around bitcoin-related conjectures is that, as time goes by and coinbase rewards are decreased, fees will make up for the incentives to continue running the blockchain. This notion is so ingrained in the bitcoin community that the bitcoin wiki literally states that "In the future, as the number of new bitcoins miners are allowed to create in each block dwindles, the fees will make up a much more important percentage of mining income." This conjecture has been shown to be false.







    share|cite|improve this answer














    share|cite|improve this answer



    share|cite|improve this answer








    edited Aug 17 at 19:51


























    community wiki





    2 revs
    dionyziz








    • 8




      I read recently someone's observation that it is extremely easy for one to conceive of cryptosystems that oneself could never conceive of breaking. Truer words have never BitToken.
      – PrimeRibeyeDeal
      Aug 17 at 17:23










    • Groan! (which is high praise for a pun)
      – Brian Hopkins
      Aug 20 at 14:15













    • 8




      I read recently someone's observation that it is extremely easy for one to conceive of cryptosystems that oneself could never conceive of breaking. Truer words have never BitToken.
      – PrimeRibeyeDeal
      Aug 17 at 17:23










    • Groan! (which is high praise for a pun)
      – Brian Hopkins
      Aug 20 at 14:15








    8




    8




    I read recently someone's observation that it is extremely easy for one to conceive of cryptosystems that oneself could never conceive of breaking. Truer words have never BitToken.
    – PrimeRibeyeDeal
    Aug 17 at 17:23




    I read recently someone's observation that it is extremely easy for one to conceive of cryptosystems that oneself could never conceive of breaking. Truer words have never BitToken.
    – PrimeRibeyeDeal
    Aug 17 at 17:23












    Groan! (which is high praise for a pun)
    – Brian Hopkins
    Aug 20 at 14:15





    Groan! (which is high praise for a pun)
    – Brian Hopkins
    Aug 20 at 14:15











    up vote
    21
    down vote













    I think the OP is looking for answers where there is a heuristic justification, apart from simply a large amount of numerical evidence, that in some instances may break down. For example, "we morally expect objects to behave a certain way, but in this instance, they don't behave quite the way we expect them to."



    Cramér's probabilistic model, from the 1930's, is a powerful heuristic for, among others, estimating gaps between prime numbers.



    However, Maier's theorem from 1985 states that, for all $lambdagt 1$,



    $$fracpi(x+(log x)^lambda)-pi(x)(log x)^lambda-1$$



    does not have a limit as $x$ goes to infinity, whereas following the Cramér heuristic, one would have a limit of $1$ for all $lambda gt 2$.



    Pintz revisits Maier's theorem, studying weaknesses of such probabilistic models with technology available in the '30s. He also hints that there are ways to improve the CM heuristic to correctly predict Maier's limit, but goes on to provide another weakness inherent to all such models that, although smaller, "seems impossible to correct."






    share|cite|improve this answer


















    • 3




      Is there an improved heuristic model that predicts Maier's theorem?
      – YCor
      Aug 17 at 21:08














    up vote
    21
    down vote













    I think the OP is looking for answers where there is a heuristic justification, apart from simply a large amount of numerical evidence, that in some instances may break down. For example, "we morally expect objects to behave a certain way, but in this instance, they don't behave quite the way we expect them to."



    Cramér's probabilistic model, from the 1930's, is a powerful heuristic for, among others, estimating gaps between prime numbers.



    However, Maier's theorem from 1985 states that, for all $lambdagt 1$,



    $$fracpi(x+(log x)^lambda)-pi(x)(log x)^lambda-1$$



    does not have a limit as $x$ goes to infinity, whereas following the Cramér heuristic, one would have a limit of $1$ for all $lambda gt 2$.



    Pintz revisits Maier's theorem, studying weaknesses of such probabilistic models with technology available in the '30s. He also hints that there are ways to improve the CM heuristic to correctly predict Maier's limit, but goes on to provide another weakness inherent to all such models that, although smaller, "seems impossible to correct."






    share|cite|improve this answer


















    • 3




      Is there an improved heuristic model that predicts Maier's theorem?
      – YCor
      Aug 17 at 21:08












    up vote
    21
    down vote










    up vote
    21
    down vote









    I think the OP is looking for answers where there is a heuristic justification, apart from simply a large amount of numerical evidence, that in some instances may break down. For example, "we morally expect objects to behave a certain way, but in this instance, they don't behave quite the way we expect them to."



    Cramér's probabilistic model, from the 1930's, is a powerful heuristic for, among others, estimating gaps between prime numbers.



    However, Maier's theorem from 1985 states that, for all $lambdagt 1$,



    $$fracpi(x+(log x)^lambda)-pi(x)(log x)^lambda-1$$



    does not have a limit as $x$ goes to infinity, whereas following the Cramér heuristic, one would have a limit of $1$ for all $lambda gt 2$.



    Pintz revisits Maier's theorem, studying weaknesses of such probabilistic models with technology available in the '30s. He also hints that there are ways to improve the CM heuristic to correctly predict Maier's limit, but goes on to provide another weakness inherent to all such models that, although smaller, "seems impossible to correct."






    share|cite|improve this answer














    I think the OP is looking for answers where there is a heuristic justification, apart from simply a large amount of numerical evidence, that in some instances may break down. For example, "we morally expect objects to behave a certain way, but in this instance, they don't behave quite the way we expect them to."



    Cramér's probabilistic model, from the 1930's, is a powerful heuristic for, among others, estimating gaps between prime numbers.



    However, Maier's theorem from 1985 states that, for all $lambdagt 1$,



    $$fracpi(x+(log x)^lambda)-pi(x)(log x)^lambda-1$$



    does not have a limit as $x$ goes to infinity, whereas following the Cramér heuristic, one would have a limit of $1$ for all $lambda gt 2$.



    Pintz revisits Maier's theorem, studying weaknesses of such probabilistic models with technology available in the '30s. He also hints that there are ways to improve the CM heuristic to correctly predict Maier's limit, but goes on to provide another weakness inherent to all such models that, although smaller, "seems impossible to correct."







    share|cite|improve this answer














    share|cite|improve this answer



    share|cite|improve this answer








    edited Aug 17 at 23:05


























    community wiki





    2 revs
    Mark S








    • 3




      Is there an improved heuristic model that predicts Maier's theorem?
      – YCor
      Aug 17 at 21:08












    • 3




      Is there an improved heuristic model that predicts Maier's theorem?
      – YCor
      Aug 17 at 21:08







    3




    3




    Is there an improved heuristic model that predicts Maier's theorem?
    – YCor
    Aug 17 at 21:08




    Is there an improved heuristic model that predicts Maier's theorem?
    – YCor
    Aug 17 at 21:08










    up vote
    18
    down vote













    It has been conjectured that, for all $n$, there is no interval of length $n$ with more primes in it than the interval between $2$ and $n+1$. You look at a table of primes, and you see how they thin out the higher up you go, and that's evidence, of a sort. But more precisely, we know that the density of the primes among the first $n$ numbers goes to zero as $n$ goes to infinity, so that seems like a heuristic supporting the conjecture.



    And the conjecture hasn't actually been disproved, but some 40 years ago, Hensley & Richards proved it contradicted the prime $k$-tuples conjecture, which has what's considered to be stronger supporting evidence. So at least one of two conjectures with heuristic support is false, we just don't have a decision yet on which one.






    share|cite|improve this answer


















    • 3




      Off topic, but I was struck by the line in the linked paper: "It was necessary to go into machine language in order to handle a sequence of 10^5 bits without taking too much computer time and memory space". How things change...
      – Yemon Choi
      Aug 18 at 17:06










    • Maybe this decreasing prime density conjecture is false just because you encounter more prime powers in [2,n+1] than in other intervals of length n : the prime powers 4, 8, 9, 16, 25, 27, 32, 49 all appear before 50 so it may not be that surprising that some translate of such an interval actually contains more primes.
      – Sylvain JULIEN
      Aug 18 at 17:33











    • And as a matter of fact, adding 100 to the previous numbers yields the three primes 109, 127 and 149.
      – Sylvain JULIEN
      Aug 18 at 17:38














    up vote
    18
    down vote













    It has been conjectured that, for all $n$, there is no interval of length $n$ with more primes in it than the interval between $2$ and $n+1$. You look at a table of primes, and you see how they thin out the higher up you go, and that's evidence, of a sort. But more precisely, we know that the density of the primes among the first $n$ numbers goes to zero as $n$ goes to infinity, so that seems like a heuristic supporting the conjecture.



    And the conjecture hasn't actually been disproved, but some 40 years ago, Hensley & Richards proved it contradicted the prime $k$-tuples conjecture, which has what's considered to be stronger supporting evidence. So at least one of two conjectures with heuristic support is false, we just don't have a decision yet on which one.






    share|cite|improve this answer


















    • 3




      Off topic, but I was struck by the line in the linked paper: "It was necessary to go into machine language in order to handle a sequence of 10^5 bits without taking too much computer time and memory space". How things change...
      – Yemon Choi
      Aug 18 at 17:06










    • Maybe this decreasing prime density conjecture is false just because you encounter more prime powers in [2,n+1] than in other intervals of length n : the prime powers 4, 8, 9, 16, 25, 27, 32, 49 all appear before 50 so it may not be that surprising that some translate of such an interval actually contains more primes.
      – Sylvain JULIEN
      Aug 18 at 17:33











    • And as a matter of fact, adding 100 to the previous numbers yields the three primes 109, 127 and 149.
      – Sylvain JULIEN
      Aug 18 at 17:38












    up vote
    18
    down vote










    up vote
    18
    down vote









    It has been conjectured that, for all $n$, there is no interval of length $n$ with more primes in it than the interval between $2$ and $n+1$. You look at a table of primes, and you see how they thin out the higher up you go, and that's evidence, of a sort. But more precisely, we know that the density of the primes among the first $n$ numbers goes to zero as $n$ goes to infinity, so that seems like a heuristic supporting the conjecture.



    And the conjecture hasn't actually been disproved, but some 40 years ago, Hensley & Richards proved it contradicted the prime $k$-tuples conjecture, which has what's considered to be stronger supporting evidence. So at least one of two conjectures with heuristic support is false, we just don't have a decision yet on which one.






    share|cite|improve this answer














    It has been conjectured that, for all $n$, there is no interval of length $n$ with more primes in it than the interval between $2$ and $n+1$. You look at a table of primes, and you see how they thin out the higher up you go, and that's evidence, of a sort. But more precisely, we know that the density of the primes among the first $n$ numbers goes to zero as $n$ goes to infinity, so that seems like a heuristic supporting the conjecture.



    And the conjecture hasn't actually been disproved, but some 40 years ago, Hensley & Richards proved it contradicted the prime $k$-tuples conjecture, which has what's considered to be stronger supporting evidence. So at least one of two conjectures with heuristic support is false, we just don't have a decision yet on which one.







    share|cite|improve this answer














    share|cite|improve this answer



    share|cite|improve this answer








    edited Aug 18 at 8:07


























    community wiki





    2 revs, 2 users 80%
    Gerry Myerson








    • 3




      Off topic, but I was struck by the line in the linked paper: "It was necessary to go into machine language in order to handle a sequence of 10^5 bits without taking too much computer time and memory space". How things change...
      – Yemon Choi
      Aug 18 at 17:06










    • Maybe this decreasing prime density conjecture is false just because you encounter more prime powers in [2,n+1] than in other intervals of length n : the prime powers 4, 8, 9, 16, 25, 27, 32, 49 all appear before 50 so it may not be that surprising that some translate of such an interval actually contains more primes.
      – Sylvain JULIEN
      Aug 18 at 17:33











    • And as a matter of fact, adding 100 to the previous numbers yields the three primes 109, 127 and 149.
      – Sylvain JULIEN
      Aug 18 at 17:38












    • 3




      Off topic, but I was struck by the line in the linked paper: "It was necessary to go into machine language in order to handle a sequence of 10^5 bits without taking too much computer time and memory space". How things change...
      – Yemon Choi
      Aug 18 at 17:06










    • Maybe this decreasing prime density conjecture is false just because you encounter more prime powers in [2,n+1] than in other intervals of length n : the prime powers 4, 8, 9, 16, 25, 27, 32, 49 all appear before 50 so it may not be that surprising that some translate of such an interval actually contains more primes.
      – Sylvain JULIEN
      Aug 18 at 17:33











    • And as a matter of fact, adding 100 to the previous numbers yields the three primes 109, 127 and 149.
      – Sylvain JULIEN
      Aug 18 at 17:38







    3




    3




    Off topic, but I was struck by the line in the linked paper: "It was necessary to go into machine language in order to handle a sequence of 10^5 bits without taking too much computer time and memory space". How things change...
    – Yemon Choi
    Aug 18 at 17:06




    Off topic, but I was struck by the line in the linked paper: "It was necessary to go into machine language in order to handle a sequence of 10^5 bits without taking too much computer time and memory space". How things change...
    – Yemon Choi
    Aug 18 at 17:06












    Maybe this decreasing prime density conjecture is false just because you encounter more prime powers in [2,n+1] than in other intervals of length n : the prime powers 4, 8, 9, 16, 25, 27, 32, 49 all appear before 50 so it may not be that surprising that some translate of such an interval actually contains more primes.
    – Sylvain JULIEN
    Aug 18 at 17:33





    Maybe this decreasing prime density conjecture is false just because you encounter more prime powers in [2,n+1] than in other intervals of length n : the prime powers 4, 8, 9, 16, 25, 27, 32, 49 all appear before 50 so it may not be that surprising that some translate of such an interval actually contains more primes.
    – Sylvain JULIEN
    Aug 18 at 17:33













    And as a matter of fact, adding 100 to the previous numbers yields the three primes 109, 127 and 149.
    – Sylvain JULIEN
    Aug 18 at 17:38




    And as a matter of fact, adding 100 to the previous numbers yields the three primes 109, 127 and 149.
    – Sylvain JULIEN
    Aug 18 at 17:38










    up vote
    13
    down vote













    Ruelle's "heuristic theory of phase transitions" (Comm. Math. Phys., Volume 53, Number 3 (1977), 195-208) turned out to be false in certain Banach spaces of interactions: see my "Generic triviality of phase diagrams in spaces of long-range interactions" (Comm. Math. Phys., Volume 106, Number 3 (1986), 459-466).






    share|cite|improve this answer


























      up vote
      13
      down vote













      Ruelle's "heuristic theory of phase transitions" (Comm. Math. Phys., Volume 53, Number 3 (1977), 195-208) turned out to be false in certain Banach spaces of interactions: see my "Generic triviality of phase diagrams in spaces of long-range interactions" (Comm. Math. Phys., Volume 106, Number 3 (1986), 459-466).






      share|cite|improve this answer
























        up vote
        13
        down vote










        up vote
        13
        down vote









        Ruelle's "heuristic theory of phase transitions" (Comm. Math. Phys., Volume 53, Number 3 (1977), 195-208) turned out to be false in certain Banach spaces of interactions: see my "Generic triviality of phase diagrams in spaces of long-range interactions" (Comm. Math. Phys., Volume 106, Number 3 (1986), 459-466).






        share|cite|improve this answer














        Ruelle's "heuristic theory of phase transitions" (Comm. Math. Phys., Volume 53, Number 3 (1977), 195-208) turned out to be false in certain Banach spaces of interactions: see my "Generic triviality of phase diagrams in spaces of long-range interactions" (Comm. Math. Phys., Volume 106, Number 3 (1986), 459-466).







        share|cite|improve this answer














        share|cite|improve this answer



        share|cite|improve this answer








        answered Aug 16 at 20:57


























        community wiki





        Robert Israel





















            up vote
            4
            down vote













            I think Hauptvermutung (the "main conjecture" in German) is a good example.
            It certainly is supported by very plausible heuristic arguments, and nobody had any doubt for half a century.






            share|cite|improve this answer






















            • What are those "very plausible heuristic arguments"?
              – Wojowu
              Aug 18 at 10:28










            • Edwin Moise in his well known paper "Affine structures ..." (Annals of Math,1952) for this sort of arguments makes a reference to "Zur Topologie der Mannigfaltigkeiten", G Nöbeling, Monatshefte für Mathematik und Physik, 1935. I can't say much more for I could not read it even if I had it (I am not very good at German). On the undergrad level, it is plausible at least for manifolds because continuous maps can be approximated by piecewise linear ones. Of course, today we know that this logic is wrong.
              – Alex Gavrilov
              Aug 18 at 11:10






            • 3




              mathoverflow.net/questions/51531/…
              – Alex Gavrilov
              Aug 18 at 11:25














            up vote
            4
            down vote













            I think Hauptvermutung (the "main conjecture" in German) is a good example.
            It certainly is supported by very plausible heuristic arguments, and nobody had any doubt for half a century.






            share|cite|improve this answer






















            • What are those "very plausible heuristic arguments"?
              – Wojowu
              Aug 18 at 10:28










            • Edwin Moise in his well known paper "Affine structures ..." (Annals of Math,1952) for this sort of arguments makes a reference to "Zur Topologie der Mannigfaltigkeiten", G Nöbeling, Monatshefte für Mathematik und Physik, 1935. I can't say much more for I could not read it even if I had it (I am not very good at German). On the undergrad level, it is plausible at least for manifolds because continuous maps can be approximated by piecewise linear ones. Of course, today we know that this logic is wrong.
              – Alex Gavrilov
              Aug 18 at 11:10






            • 3




              mathoverflow.net/questions/51531/…
              – Alex Gavrilov
              Aug 18 at 11:25












            up vote
            4
            down vote










            up vote
            4
            down vote









            I think Hauptvermutung (the "main conjecture" in German) is a good example.
            It certainly is supported by very plausible heuristic arguments, and nobody had any doubt for half a century.






            share|cite|improve this answer














            I think Hauptvermutung (the "main conjecture" in German) is a good example.
            It certainly is supported by very plausible heuristic arguments, and nobody had any doubt for half a century.







            share|cite|improve this answer














            share|cite|improve this answer



            share|cite|improve this answer








            edited Aug 18 at 12:03


























            community wiki





            2 revs, 2 users 67%
            Alex Gavrilov












            • What are those "very plausible heuristic arguments"?
              – Wojowu
              Aug 18 at 10:28










            • Edwin Moise in his well known paper "Affine structures ..." (Annals of Math,1952) for this sort of arguments makes a reference to "Zur Topologie der Mannigfaltigkeiten", G Nöbeling, Monatshefte für Mathematik und Physik, 1935. I can't say much more for I could not read it even if I had it (I am not very good at German). On the undergrad level, it is plausible at least for manifolds because continuous maps can be approximated by piecewise linear ones. Of course, today we know that this logic is wrong.
              – Alex Gavrilov
              Aug 18 at 11:10






            • 3




              mathoverflow.net/questions/51531/…
              – Alex Gavrilov
              Aug 18 at 11:25
















            • What are those "very plausible heuristic arguments"?
              – Wojowu
              Aug 18 at 10:28










            • Edwin Moise in his well known paper "Affine structures ..." (Annals of Math,1952) for this sort of arguments makes a reference to "Zur Topologie der Mannigfaltigkeiten", G Nöbeling, Monatshefte für Mathematik und Physik, 1935. I can't say much more for I could not read it even if I had it (I am not very good at German). On the undergrad level, it is plausible at least for manifolds because continuous maps can be approximated by piecewise linear ones. Of course, today we know that this logic is wrong.
              – Alex Gavrilov
              Aug 18 at 11:10






            • 3




              mathoverflow.net/questions/51531/…
              – Alex Gavrilov
              Aug 18 at 11:25















            What are those "very plausible heuristic arguments"?
            – Wojowu
            Aug 18 at 10:28




            What are those "very plausible heuristic arguments"?
            – Wojowu
            Aug 18 at 10:28












            Edwin Moise in his well known paper "Affine structures ..." (Annals of Math,1952) for this sort of arguments makes a reference to "Zur Topologie der Mannigfaltigkeiten", G Nöbeling, Monatshefte für Mathematik und Physik, 1935. I can't say much more for I could not read it even if I had it (I am not very good at German). On the undergrad level, it is plausible at least for manifolds because continuous maps can be approximated by piecewise linear ones. Of course, today we know that this logic is wrong.
            – Alex Gavrilov
            Aug 18 at 11:10




            Edwin Moise in his well known paper "Affine structures ..." (Annals of Math,1952) for this sort of arguments makes a reference to "Zur Topologie der Mannigfaltigkeiten", G Nöbeling, Monatshefte für Mathematik und Physik, 1935. I can't say much more for I could not read it even if I had it (I am not very good at German). On the undergrad level, it is plausible at least for manifolds because continuous maps can be approximated by piecewise linear ones. Of course, today we know that this logic is wrong.
            – Alex Gavrilov
            Aug 18 at 11:10




            3




            3




            mathoverflow.net/questions/51531/…
            – Alex Gavrilov
            Aug 18 at 11:25




            mathoverflow.net/questions/51531/…
            – Alex Gavrilov
            Aug 18 at 11:25

















             

            draft saved


            draft discarded















































             


            draft saved


            draft discarded














            StackExchange.ready(
            function ()
            StackExchange.openid.initPostLogin('.new-post-login', 'https%3a%2f%2fmathoverflow.net%2fquestions%2f308464%2fare-there-examples-of-conjectures-supported-by-heuristic-arguments-that-have-bee%23new-answer', 'question_page');

            );

            Post as a guest













































































            Popular posts from this blog

            How to check contact read email or not when send email to Individual?

            Bahrain

            Postfix configuration issue with fips on centos 7; mailgun relay